You are on page 1of 12

1

MATH 135 SOS Review Package


Induction
Mathematical induction is a proof technique used to prove statements that depend on a natural number n, i.e. statements of the form n P, P (n) where P (n) is some statement about n. We review the statements of the Principle of Mathematical Induction and the Principle of Strong Induction below. Theorem (Principle of Mathematical Induction). Let P (n) be a statement about n P. If 1. P (1) is true 2. and if for each k P, P (k) is true implies P (k + 1) is true then P (n) is true for all n P. Theorem (Principle of Strong Induction). Let P (n) be a statement about n P. If 1. P (1) is true 2. and if for each k P, P (1), . . . , P (k) are true implies P (k + 1) is true then P (n) is true for all n P. The main dierence between induction and strong induction is in the second hypothesis. In induction, we assume only that P (k) is true and we use this to prove P (k + 1) is true, whereas in strong induction, we assume P (1), . . . , P (k) are true and we use this to prove P (k + 1). We separate a proof by induction into three steps. First we verify P (1) or some other base case(s). Then we make an induction hypothesis. If we are using induction, we assume P (k) is true. If we are using strong induction, we assume P (1), , P (k) are true. Finally, in the induction conclusion, we use our induction hypothesis to prove P (k + 1). We give three examples to illustrate typical uses of induction. Example. Prove that for each n 2, n P
n

1+
r=2

r2

1 1

2n . n+1

Solution. Let P (n) be the statement

n r=2

1+

1 r2 1

Base Case: We verify P (2) is true. P (2) is the statement


2

2n n+1 . We 2 r=2 1

prove the result by induction on n. +


1 r2 1

= 4 . We have 3

1+
r=2

r2

1 1

=1+

1 4 = 3 3

which veries P (2).

Induction Hypothesis: Assume P (k) is true for some k 2, k P. That is


k

1+
r=2

r2

1 1

2k . k+1

2 Induction Conclusion: We verify P (k + 1) is true. P (k + 1) is the statement


k+1

1+
r=2

1 r2 1

2(k + 1) . (k + 1) + 1

We have
k+1

1+
r=2

1 21 r

=
r=2

1+

r2

1 1

1+

1 (k + 1)2 1 using the induction hypothesis

2k 1 1+ k+1 (k + 1)2 1 2k k 2 + 2k + 1 = k+1 k(k + 2) 2k (k + 1)2 = k + 1 k(k + 2) 2(k + 1) = (k + 1) + 1

which veries P (k + 1). Therefore the result follows by the Principle of Mathematical Induction. Example. A sequence {xn } is dened by x1 = 0 x2 = 30 xn = xn1 + 6xn2 for n 3 Prove that for each n P xn = 2 3n + 3 (2)n . Solution. Let P (n) be the statement xn = 2 3n + 3 (2)n . We prove the result by strong induction on n. Base Case: We verify P (1) and P (2). P (1) is the statement x1 = 2 31 + 3 (2)1 . We have x1 = 0 and 2 31 + 3 (2)1 = 0 so P (1) is true. P (2) is the statement x2 = 2 32 + 3 (2)2 . We have x2 = 30 and 2 32 + 3 (2)2 = 18 + 12 = 30 so P (2) is true. Induction Hypothesis: Assume P (1), P (k) are true for some k 2, k P. That is xi = 2 3i + 3 (2)i for 1 i k. Induction Conclusion: We verify P (k + 1) is true. P (k + 1) is the statement xk+1 = 2 3k+1 + 3 (2)k+1 . We have xk+1 = xk + 6xk1 by the recursive denition of {xn } = (2 3k + 3 (2)k ) + 6(2 3k1 + 3 (2)k1 ) by the induction hypothesis = 2 3k + 3 (2)k + 4 3k + 32 2 (2)k1 = (2 + 4) 3k + (6 + 18)(2)k1 = 2 3k+1 + 3 (2)2 (2)k1 = 2 3k+1 + 3 (2)k+1 which veries P (k + 1). Therefore the result follows by the Principle of Strong Induction.

3 Example. Prove that a set with n elements has 2n subsets. Proof. Solution Let P (n) be the statement: A set with n elements has 2n subsets. We prove the result by induction. Base Case: We verify P (1). P (1) is the statement A set with 1 element has 21 subsets. Let S be a set with 1 element. The subsets of S are and S. Thus S has 2 = 21 subsets which veries P (1). Induction Hypothesis: Suppose P (k) is true for some k P. That is suppose a set with k elements has 2k subsets. Induction Conclusion: We verify P (k + 1). P (k + 1) is the statement A set with (k + 1) elements has 2k+1 subsets. Let S be a set with (k + 1) elements and let x S. Then S {x} is a set with k elements so by induction hypothesis it has 2k subsets. Let T be a subset of S. Either x T or x T . If x T then / T {x} is a subset of S {x}. So we get 2k such subsets T by the induction hypothesis. If x S, then / k such subsets T by the induction hypothesis. Thus in total we get T is a subset of S {x}. So we get 2 2k + 2k = 2 2k = 2k+1 subsets. Exercise. Let S be a non-empty subset of P. Show that S has a least element. (that is, show that there exists x S such that for all y S we have x y.) Solution. We prove the result by contradiction. Suppose there exists a non-empty subset S of P which does not have a least element. We consider the set T dened by T = P \ S = {x P | x S}. / We show that T = P by strong induction. Let P (n) be the statement n T . Base Case: We verify P (1). P (1) is the statement 1 T . Suppose for a contradiction that 1 T . Then / 1 S. But then since for each x P we have 1 x so we have found a least element for S. This is a contradiction and so we must have 1 T . Induction Hypothesis: Suppose that P (1), . . . , P (k) are true for some k P. Induction Conclusion. We verify P (k + 1) which is the statement k + 1 T . Suppose for a contradiction that k + 1 T . Then k + 1 S. But since 1, 2, , k S then k + 1 is a least element for S which is a / / contradiction. Thus we must have k + 1 T . Therefore by strong induction we have n T for each n P, i.e. T = P. This implies S = which is a contradiction. Thus S must have a least element.

Divisibility (Again...)
We quickly review the material covered before the midterm. Denition. Let a and b be integers. We say a divides b and we write a | b if there exists an integer k such that b = ka. Denition. Let a and b be integers, both not zero and let d be a positive integer. We say that d is the greatest common divisor of a and b and we write gcd(a, b) = d if: 1. d | a and d | b 2. if c is an integer such that c | a and c | b, then c d.

4 We now recall several propositions and theorems covered before the midterm. 1. Transitivity of Divisibility. If a | b and b | c, then a | c. 2. Divisibility of Integer Combinations. If a | b and a | c, then a | bx + cy for any x, y Z. 3. Bounds by Divisibility. a | b and b = 0, then |a| |b|. 4. GCD with Remainder. Let a and b be integers, both not zero. If a = qb + r for some integers q and r, then gcd(a, b) = gcd(b, r). 5. GCD Certicate Theorem. Let d be a positive integer and let a and b be integers. If d | a, d | b, and there exist integers x and y such that ax + by = d, then d = gcd(a, b). 6. Extended Euclidean Algorithm. Let a and b be integers and let d be a positive integer. If d = gcd(a, b), then there exist integers x and y such that ax + by = d. 7. Linear Diophantine Equation Theorem 1. The linear Diophantine equation ax + by = c has solution if and only if gcd(a, b) | c. 8. Linear Diophantine Equation Theorem 2. If gcd(a, b) = d = 0 and x = x0 , y = y0 is a particular solution , then the complete integer solution is b a x = x 0 + n , y = y0 n d d for all n Z. We give two examples to review some of the ideas. Example (Practice Problem 5). Let a, b, c Z. Prove that if a and c are coprime, then gcd(ab, c) = gcd(b, c). Solution. Let d = gcd(b, c). We show d = gcd(ab, c). Since gcd(a, c) = 1, then there exists x, y Z such that ax+cy = 1 by the Extended Euclidean Algorithm. Multiplying this by ab we get b(a2 x)+c(abx) = ab. Since d | b and d | c we get d | b(a2 x) + c(aby) = ab by Divisibility of Integer Combinations. By denition we have d | c. Thus d is common divisor of ab and c. Suppose e | ab and e | c. We multiply ax + cy = 1 by b to obtain ab(x) + c(by) = b. Since e | ab and e | c then e | ab(x) + c(by) = b. Thus e is a common divisor of b and c. Therefore we conclude e d as gcd(b, c). This shows d = gcd(ab, c). Example. Let k P. Show that that there exists k consecutive composite integers. Solution. Let n = k + 1 and consider the k consecutive integers n! + 2, n! + 3, , n! + (k + 1). For each 2 i k + 1 = n, we know that i | n! because i n and i | i so i | n! + i by Divisibility of Integer Cominbations. Since i = 1 and i = n! + i this shows n! + i is composite. Thus we have found k consecutive composite integers.

Another way to think about divisibility


We saw another way to think about divisibility in terms of prime factorizations. First, we recall that we recall that we can always factor a positive integer into primes. Theorem. Let x P. Then x can be written as a product of primes which is unique up to re-ordering. That is, we can write x = pr1 prk where p1 , , pk are distinct primes and r1 , , rk P. Moreover if 1 k
s x = pr1 prk = q11 qlsl 1 k

are two prime factorizations, then k = l and after re-ordering pi = qi for each 1 i k. The next proposition connects divisibility and prime factorizations. Proposition. If x = pr1 prk is a prime factorization of x, then d is a positive divisor of x if and only 1 k d = ps1 psk where for each 1 i k we have 0 si ri . 1 k We also get a formula for the greatest common divisor of x and y as follows: suppose x = pr1 prk and 1 k y = ps1 psk where p1 , , pk are distinct primes and r1 , , rk , s1 , , sk 0. Then 1 k gcd(x, y) = pe1 pek 1 k where ei = min(ri , si ). Example. How many positive divisors does 6696 have? Solution. We have 6696 = 23 33 31. By our proposition, d is a positive divisor of 6696 if and only if d = 2d1 3d2 31d3 where 0 d1 3, 0 d2 3, and 0 d3 1. Thus there are 4 4 2 = 32 positive divisors. Example. Find gcd(12!, 211 34 57 ). Solution. It is enough to just count the number of factors of 2,3, and 5 in 12! = 12 11 10 9 8 7 6 5 4 3 2 1. We get 10 factors of 2, 5 factors of 3, and 2 factors of 5. Thus gcd(12!, 211 34 57 ) = 210 34 52 .

Congruences
Denition. Let m be a xed positive integer. Let a, b Z. We say a is congruent to b modulo m and we write a b mod m if m | a b. We have a a + m a + 2m a + 3m a + km mod m so we can intuitively think of as treating integer multiples of m as zero. We have the following basic properties of the congruence relation. mod m

6 Proposition (Congruence is an Equivalence Relation). Let m be a xed positive integer. Let a, b, c Z. Then 1. a a mod m [identity] 2. if a b mod m then b a mod m [reexive] 3. if a b mod m and b c mod m then a c mod m [transitive]. Proposition (Properties of Congruence). Let m be a xed positive integer. Let a, b, a , b Z and let k P. If a a mod m and b b mod m, then 1. a + b a + b mod m 2. a b a b mod m 3. ab a b mod m 4. ak (a )k mod m What about division? In R, we know if ac = bc and c = 0 then a = b. (Another way to say this is that every non-zero real number has a multiplicative inverse.) But when we consider the integers modulo 4, we know 5 2 7 2 mod 4 since 10 14 mod 4 but 5 7 mod 4. When can we cancel out or divide common factors? Proposition (Congruence and Division). Let m be a xed positive integer. Let a, b, c Z. If ac bc mod m and gcd(c, m) = 1 then a b mod m. We have now proven several propositions about the congruence relation. However we still do not fully understand the relation. The following proposition tells us that the congruence relation is the same as the remainder when divided by m relation. Proposition (Congruent I Same Remainder). Let m be a xed positive integer. Let a, b Z. Then a b if and only if a and b have the same remainder when divided by m. Proof. By the Division Algorithm we have a = q1 m + r1 where q1 , r1 Z and 0 r1 < m b = q2 m + r2 where q2 , r2 Z and 0 r2 < m

(=). Suppose a b mod m. We must show r1 = r2 . Since a b mod m then m | a b. We see that a b = (q1 q2 )m + (r1 r2 ). Thus we have m | (a b) (q1 q2 )m = r1 r2 by Divisibility of Integer Combinations. On the other hand we have m < r1 r2 < m. Since the only integer dividing m strictly between m and m is 0, we must have r1 r2 = 0 or r1 = r2 . (=). Suppose r1 = r2 . Then a b = (q1 q2 )m and thus m | a b so a b mod m. We can use this proposition to nd remainders (without actually performing the division!) and to prove tests for divisibility. Example. What is the last digit of 333 77 ? Solution. The last digit is the same the remainder when we divide by 10. Instead of performing the division, we can simply compute 333 77 mod 10. We notice 32 = 9 1 mod 10 so that 333 = (32 )16 3 (1)16 3 3 mod 10.

7 Similarly we have 72 = 49 1 mod 10 so that 77 = (72 )3 7 (1)3 7 3 Therefore 333 77 3 3 9 so the last digit of 333 77 is 9. We have one last extremely useful result about congruences. Theorem (Fermats Little Theorem). Let p Z be a prime. If a is an integer such that p does not divide a then ap1 1 mod p. This theorem is very handy when dealing with prime moduli. We give a typical example. Example. Suppose that n Z. Prove that n91 n7 mod 91. Solution. We want to apply Fermats Little Theorem so we must turn the problem into one about primes. We observe 91 = 7 13 so instead we try and show n91 n7 n n
7 7

mod 10.

mod 10

mod 7 mod 13.

(1) (2)

First we prove a claim which says that we can do this. (We could alternatively appeal to the Chinese Remainder Theorem which we review below.) Claim. Let x Z and let p, q Z be distinct primes. If p | x and q | x then pq | x. Verication. Since p and q are distinct, gcd(p, q) = 1 so by the Extended Euclidean Algorithm there exist u, v Z such that pu + qv = 1. Multiplying this by x we obtain pux + qvx = x. Now p | x and q | so there exists integers m and n such that x = mp and x = nq. Thus we have x = pu(nq) + qv(mp) = pq(un + vm) which implies pq | x as required. Thus if n91 n7 mod 7 and n91 n7 mod 13 then 7 | n91 n7 and 13 | n91 n7 so by our claim 91 | n91 n7 which is equivalent to n91 n7 mod 91. To verify the rst equation, we consider two cases. If n 0 mod 7 then we have n91 n7 0 mod 7. Otherwise, we suppose n 0 mod 7. Then we may apply Fermats Little Theorem to obtain n7 n mod 7 which implies n91 = (n7 )13 n13 n6 n7 n7 mod 7. Similarly we have n91 n7 mod 13. Thus n91 n7 mod 91.

The integers modulo m: Zm


Denition. Let m be a xed positive integer. Let a Z. The congruence class of a modulo m is the set [a]m = {x Z | x a mod m} For example if m = 6, we have [5]6 = { , 7, 1, 5, 11, 17, } = [11]6 . Intuitively we have [5]6 is all the integers which have remainder 5 when we divide by 6. We dene Zm = {[0]m , , [m 1]m }. We dene addition and multiplication in Zm as follows [a]m + [b]m = [a + b]m [a]m [b]m = [ab]m .

We saw Zm does not have the same algebraic properties as Z. For example, in Z6 , 2 3 = 0 (i.e. 2 3 0 mod 6), but neither 2 nor 3 is 0 in Z6 ! Also, in Zp , for p prime, we saw that every non-zero element has a multiplicative inverse. But in Z only 1 and 1 have multiplicative inverses.

Congruences
Denition. Let m be a positive integer. Let a, c Z. A linear congruence in the variable x is a relation of the form ax c mod m. We notice ax c mod m m | ax c y Z s.t. ax c = my y Z s.t. ax + my = c Similar to our theorem about linear diophantine equations we have the following theorem about linear congruences. Theorem (Linear Congruence Theorem). Let gcd(a, m) = 1. The linear congruence ax c mod m has a solution if and only if d | c. Moreover, if x = x0 is a particular solution then the complete solution is x x0 or x x0 , x0 + mod m d

m m , , x0 + (d 1) . d d

9 Example. Solve the congruence 21x 12 mod 15 Solution. Since gcd(21, 15) = 3 and 3 | 12 the congruence has 3 solutions. To nd the solutions we solve the linear Diophantine equation 21x + 15y = 12. We get 21 = 1 15 + 6 15 = 2 6 + 3 6=23+0 so q3 = 1 and q4 = q5 = 2. Next we use the Extended Euclidean Algorithm to get x 1 0 1 -2 5 y 0 1 -1 3 -7 ri 21 15 6 3 0 qi 0 0 1 2 2

Thus 21(2) + 15(3) = 3. Multiplying by 4, we get 21(8) + 15(12) = 12 so a particular solution is x = 8, y = 12. Thus a particular solution to the congruence is x 8 7 The complete solution is x 7, 7 + 15 15 , 7 + 2 2, 7, 12 3 3 mod 15. mod 15.

We also have the following result for solving systems of linear congruences. Theorem (Chinese Remainder Theorem). Let a1 , a2 , m1 , m2 Z with gcd(m1 , m2 ) = d. Consider the system S x a1 x a2 mod m1 mod m2

of linear congruences. If d does not divide a2 a1 then S has no solutions. If d | a2 a1 then the complete solution is given by m1 m2 x x0 mod d where x0 is a particular solution to S.

10 Example. Solve the system of congruences x2 x5 mod 7 mod 8.

Solution. Since x satises 1 we may write x = 2 + 7y for some y Z and we substitute this into 2 to obtain 7y 3 mod 8.

We could solve this linear congruence as above, however, we present an alternative solution. We notice 72 1 mod 8 so that y 21 5 mod 8. Thus we may write y = 5 + 8z for some z Z. Substituting this back into x = 2 + 7y, we get x = 37 + 56z or x 37 mod 56.

Example. Solve the congruence x5 + x3 2 mod 5.

Solution. By Fermats Little Theorem we know that x5 x mod 5 so we can instead solve the polynomial congruence x3 + x 2 mod 5. We make a table. x x3 3+x x Thus the solution is x 2 mod 5. 0 0 0 1 1 2 2 3 0 3 2 0 4 4 3

Complex Numbers
Denition. We dene C to be the set {x + iy | x, y R} where i2 = 1. Let z = x + iy C. The real part of z is Re(z) = x. The imaginary part of z is Im(z) = y. The conjugate of z is z = x iy. The norm of z is |z| = x2 + y 2 . We can dene addition, subtraction, multiplication, and division of complex numbers in the natural way and they have the same properties as addition, subtraction, multiplication, and division of real numbers. (see your notes!) We can also represent z = x + iy as z = r(cos + i sin ) which is called the polar representation of z.

11 Example. Write z = 1 + i in polar form. Solution. Sketching the point (1, 1) we see that r = z= 2 cos 1+1= 2 and = . Thus 4

+ i sin . 4 4

The polar representation of z gives us a nice way of performing exponentiation. Theorem. Let z = r(cos + i sin ) C and let n Z. Then z n = rn (cos n + i sin n). Example. Prove cos 3 = 4 cos3 3 cos sin 3 = 3 sin 4 sin3 . Solution. We consider z = cos + i sin and compute z 3 in two ways. On the one hand, z 3 = (cos + i sin )3
3

=
i=0 3

3 (cos )3i (i sin )i i

= cos + 3i cos2 sin 3 cos sin2 i sin3 = (cos3 3 cos sin2 ) + i(3 cos2 sin sin3 ). Whereas, using the theorem above, we have z 3 = cos 3 + i sin 3. Equating real and imaginary parts, we get cos 3 = cos3 3 cos sin2 = cos3 3 cos (1 cos2 ) = 4 cos3 3 cos sin 3 = 3 cos2 sin sin3 = 3(1 sin2 ) sin sin3 = 3 sin 4 sin3 .

Example. Find a real cubic polynomial with roots 1 and i. Solution. We know that if f is a real polynomial and z is a complex root of f then z is also a root of f . Thus we see f (x) = (x i)(x + i)(x 1) = (x2 + 1)(x 1) = x3 x2 + x 1 is a real cubic polynomial with roots 1 and i.

12 Example. Prove that if |z| = 1 or |w| = 1 then |z w| = |1 zw|. Solution. We have |1 zw|2 = (1 zw)(1 zw) = (1 zw)(1 zw) = 1 zw zw + zzww

Suppose |z| = 1. Then zz = 1 and so we get |1 zw|2 = 1 zw zw + zzww = zz zw zw + ww = |z w|2 and thus |z w| = |1 zw|. The case |w| = 1 is similar.

You might also like